LSAT and Law School Admissions Forum

Get expert LSAT preparation and law school admissions advice from PowerScore Test Preparation.

User avatar
 Dave Killoran
PowerScore Staff
  • PowerScore Staff
  • Posts: 5852
  • Joined: Mar 25, 2011
|
#26535
Complete Question Explanation

The correct answer choice is (B)

From the Not Laws we know that neither L or M can fill the first position, and we also know F must be second. Thus, the remaining pieces—G, H, J, and K—could be first. Answer choice (B) is thus correct.
 cotero
  • Posts: 1
  • Joined: Jul 16, 2016
|
#31174
I'm not understanding the simple explanation.

As L or M cannot go in the first position and F has to go second, why does answer A, C or E not work? I understand why D does not work, because H has to go before L.

This is how I'm doing the lineup:

Answer A: G, J, K
Line up: G, F, J, K, H, L, M

Answer C:G, H, J, L
Line up: G, F, H, J, L, K, M

Answer E: H, J, K, L, M
Line up: H, F, J, K, L, M, G
 Adam Tyson
PowerScore Staff
  • PowerScore Staff
  • Posts: 5153
  • Joined: Apr 14, 2011
|
#31191
Thanks for the question, cotero! I think you may be misunderstanding what they are asking here, and you aren't alone. A lot of folks make the same mistake the first time they encounter a question like this one.

The question is not asking which answer could be an acceptable order of those variables. If they were, several answers would work as you pointed out.

Instead, the question is asking for a list of all the variables that could, in some solution, be in position #1. F is always in space #2, so that cannot be on the list because it is never in space #1.

We know from the first question, the list question, that G can be in the first space, so G has to be included in the correct answer. That eliminates answer E. We know from the H-L-M sequence that neither L nor M can ever be in the first space, so that eliminates answers D and E because they include L in the list. That leaves you with answers A and B, and the only difference between them is H. If H can go first in even one solution, then B has to be the right answer, and if it can never go first then A has to be right.

There's no problem with H going first, because then it will absolutely be ahead of L and M and it will not be next to G. One possible solution with H first is HFLMGJK, for example. That proves that B is the right answer - G, H, J and K are all the variables that could ever be in the first position; F, L and M are the only ones that can never go first.

I hope that clears it up, and that it helps you better prepare for similar questions down the road!

Get the most out of your LSAT Prep Plus subscription.

Analyze and track your performance with our Testing and Analytics Package.